Riemann hypothesis equivalences,Robin inequality,Lagarias criterion, and Riemann hypothesis

23 January 2024, Version 1
This content is an early or alternative research output and has not been peer-reviewed by Cambridge University Press at the time of posting.

Abstract

In this paper, we briefly review most of accomplished research in Riemann Zeta function and Riemann hypothesis since Riemann's age including Riemann hypothesis equivalences as well. We then make use of Robin and Lagarias' criteria to prove Riemann hypothesis. The goal is, using Lagarias criterion for $n\geq 1$ since Lagarias criterion states that Riemann hypothesis holds if and only if the inequality $\sum_{d|n}d\leq H_{n}+\exp(H_{n})\log(H_{n})$ holds for all $n\geq 1$. Although, Robin's criterion is used as well. Our approach breaks up the set of the natural numbers into three main subsets. The first subset is $\{n\in \mathbb{N}| ~ 1\leq n\leq 5040\}$. The second one is $\{n\in \mathbb{N}| ~ 5041\leq n\leq 19685\}$ and the third one is $\{n\in \mathbb{N}| ~ n\geq 19686\}$. In our proof, the third subset for even integers is broken up into odd integer class number sets. Then, mathematical arguments are stated for each odd integer class number set. Odd integer class number set is introduced in this paper. Since the Lagarias criterion holds for the first subset regarding computer aided computations, we do prove it using both Lagarias and Robin's criteria for the second and third subsets and mathematical arguments accompanied by a large volume of computer language programs. It then follows that Riemann hypothesis holds as well.

Keywords

Elementary number theory
Analytic number theory
Sum of divisors function
Robin's criterion
Lagarias' criterion
Odd integer class number set

Comments

Comments are not moderated before they are posted, but they can be removed by the site moderators if they are found to be in contravention of our Commenting and Discussion Policy [opens in a new tab] - please read this policy before you post. Comments should be used for scholarly discussion of the content in question. You can find more information about how to use the commenting feature here [opens in a new tab] .
This site is protected by reCAPTCHA and the Google Privacy Policy [opens in a new tab] and Terms of Service [opens in a new tab] apply.
Comment number 4, Ahmad Sabihi: Mar 12, 2024, 21:27

Your counter example is completely wrong. You should consider it in this format: f (x)=sin x^2=O (x^2) and g (x)=x^2, then we have lim |O (x^2)|/x^2 <A when x tends to infinity. This means that |sin x^2|<Ax^2 and since A>=1,then the inequality is true. Just differentiating the inequality for sin x^2>0, we find the inequality 2xcosx^2<2Ax, where is true and means that |f'(x)|<Ag'(x) or f'(x)=O (g'(x)) and the result is complete. If you could not be convinced with my arguments, I will not be able to bring additional clarifications. But you should be aware that this paper has passed peer review process in some prestigious journals and reviewed by many distinguished mathematicians. None of them claimed that this proposition is wrong. I respect to your opinion, but I have no additional time to discuss with you. You are able to whether accepting my arguments or not...decision is yours. Have a nice weekend. I will not respond to your other comments.

Comment number 3, Maximilian HASLER: Mar 12, 2024, 15:41

I do know very whell what the O(.) and o(.) notations mean. But you are wrong. You give 2 examples where your proposition holds, but you ignore my simple counter-example of f(x)=sin x² => f'(x) = 2x cos x which is an unbounded function and does not satisfy f' = O(g') with g'=1. The result being wrong, the proof is obviously also wrong. In particular, you incorrectly assume that the logical negation of " f < A g " is " f >= A g ", which is not the case. The former is false as soon as there is *one point* (in the considered domain) in which it does not hold. For example, " sin < 0 " is wrong, but "sin >= 0" is not true, either.

Comment number 2, Ahmad Sabihi: Mar 12, 2024, 11:07

Thank you for your comment, although you misunderstood mean of the proposition. We mean of f(x)=O(g(x)) is not a composition of two functions as ordinary analysis. If you accurately review the proposition and its proof, you will understand we mean lim |f(x)|/g(x)<A, when x tends to infinity where A is a positive constant. For example let f(x)=x and g(x)=x, then lim |x|/x<A, when x tends to infinity where A>=1. Then |x|<Ax, just differentiating the both sides of the inequality for x>0 gives us 1<A. The other example: suppose f(x)=sinx=O(g(x)) and g(x)=x, we know sinx/x<1 for x>0, then |O(g(x))| /x <A and A>=1, then |O(g(x))|=|sin x|< Ax, differentiating both sides of the inequality for x>0 and Sinx >0 gives us cos x<A, where is true or for sinx <0, we have again -sinx<A. I hope these help you to understand proposition.

Comment number 1, Maximilian HASLER: Mar 11, 2024, 05:46

Proposition 1 is wrong. It claims that if g >= 0, g' > 0, then f=O(g) => f'=O(g'). But a simple counter example is g(x)=x and f(x)=sin(x²)=O(g) (even o(g)) and g' = 1 > 0 but f'(x) = 2 x cos(x²) is not O(g') = O(1).